Beweis, dass die durch den Leiteroperator gegebenen Energiezustände eines harmonischen Oszillators alle Zustände enthalten

In der Quantenmechanik lernte ich beim Studium des harmonischen Oszillators Leiteroperatoren kennen. Und mir wurde klar, dass Sie, wenn Sie in der Lage sind, einen beliebigen Energiezustand des harmonischen Quantenoszillators zu finden oder zu bestimmen, mit den Leiteroperatoren auch die anderen Energiezustände bestimmen können. In keinem der Texte, die ich gelesen habe, habe ich jedoch die folgende Tatsache gefunden:

Die durch das obige Verfahren, dh unter Verwendung des Kettenoperators, bestimmten Energiezustände sind die einzig möglichen Energiezustände des harmonischen Oszillators. Es gibt keinen Energiezustand, der nicht durch den Leiteroperator gegeben ist.

Das ist also meine Frage:

Sind die vom Leiteroperator bestimmten Energiezustände im Falle eines harmonischen Oszillators die einzig möglichen Energiezustände? Ist jeder andere Energiezustand möglich? Und was ist der Beweis?

Es ist allgemein bekannt, dass die oben genannten Zustände die einzig möglichen Energiezustände sind, aber ich möchte einen rigorosen Beweis dafür, dass kein anderer Zustand möglich ist. Das in verschiedenen Büchern erwähnte Analyseverfahren zeigt jedoch eine Näherungslösung der Schrödinger-Gleichung, die in dem Sinne als streng angesehen werden kann, dass sie die Gleichung löst, um die Lösungen abzuleiten, obwohl sie bestimmte Näherungen berücksichtigt.

Ich habe nach dieser Antwort in Büchern über Quantenmechanik gesucht, die von DJ Griffiths, Gasiorowicz, Dirac und Online-Ressourcen wie OCW, den Kursen der University of Columbia, caltech.edu geschrieben wurden, aber ich konnte keine richtige Antwort finden.

Ich bin mir nicht sicher, was Sie meinen, wenn Sie sagen, dass das analytische Verfahren eine Näherungslösung ist. Beziehen Sie sich auf die asymptotische Analyse, in der Sie Ihre Wellenfunktion schreiben als ψ ( x ) e a x 2 im Großen und Ganzen x und dann definieren ψ ( x ) = f ( x ) e a x 2 ? Dies ist keine Annäherung, sondern nur eine Möglichkeit, Ihre Gleichung in Form einer schöneren Funktion neu zu definieren f ( x ) .
@BobKnighton ja, ich spreche von der asymptotischen Analyse und so viel ich von Griffiths lernen konnte, sieht es ein bisschen wie eine Annäherung aus. Es ist nicht gerade das, was man mathematisch streng nennt.
Es ist sehr viel keine Annäherung. Die gegebene Lösung ist exakt, und der Weg dorthin ist rigoros. Der Teil, der wie eine Annäherung aussieht (die asymptotische Analyse), ist einfach eine Möglichkeit, die folgenden Ergebnisse zu motivieren. Wir könnten dasselbe ohne die Analyse tun, indem wir sagen: „Definiere eine neue Funktion f ( x ) = ψ ( x ) / e a x 2 und schreiben Sie die Schrödinger-Gleichung in Bezug auf f . Dann können wir das zeigen f muss ein abschließendes Polynom sein, wenn ψ ist zu begrenzen." Sie ist absolut streng und äquivalent (aber nur ein bisschen weniger elegant als) die Operatormethode.
@BobKnighton Was ist die Notwendigkeit, eine neue Funktion zu definieren, während eine Differentialgleichung gelöst wird? Wenn wir also eine neue Funktion definieren, müssen wir angeben, dass die Lösungen der ursprünglichen Gleichung alle in den Lösungen der modifizierten Gleichung enthalten sind. Daher gibt es einen gewissen Mangel an Strenge bei der Lösung von Differentialgleichungen durch asymptotische Analyse. Die gegebene Funktion hat eine Exponentialform und obwohl dies physikalisch gerechtfertigt ist, ist aus mathematischer Sicht viel mehr Strenge erforderlich.
Ich bin mir nicht ganz sicher, ob ich verstehe, was Sie fragen. Wir definieren eine neue Funktion, um die Gleichung leichter lösbar zu machen. Es macht nämlich die neue Gleichung durch Potenzreihenmethoden lösbar (während die ursprüngliche Gleichung dies nicht ist, da die Lösungen zerfallen müssen). Und die Lösungen der ursprünglichen Gleichung sind sicherlich in den Lösungen der neuen Gleichung enthalten. Wenn wir eine Lösung finden für f , dann haben wir automatisch ψ ( x ) = f ( x ) e a x 2 . Es ist keine Strenge mehr erforderlich, da jede durchgeführte Operation genau definiert ist.
In der Tat, wenn Sie die Lösungen, die Sie beispielsweise in Griffiths gefunden haben, in Bezug auf Hermite-Polynome nehmen, werden Sie feststellen, dass sie genau die Eigenfunktionen des Hamilton-Operators sind. Es wurde keine einzige Annäherung vorgenommen.
Zugehöriges/mögliches Duplikat: physical.stackexchange.com/q/23028/50583
Ich bin sicher, dass es wahrscheinlich eine nette Lösung der Darstellungstheorie für dieses Problem gibt: Eigenzustände von H sollten sich zu einer irreduziblen Darstellung V der Symmetriegruppe des Modells anordnen. V beinhaltet das Vakuum. Aber auch die durch Einwirkung auf das Vakuum mit den Leiteroperatoren gebildete Darstellung U ist irreduzibel. Also U=V. Ich habe das aber noch nicht ganz durchdacht, nur ein paar kurze Gedanken.

Antworten (5)

Das ist eine fantastische Frage! Lass uns anfangen.

Ich gehe davon aus, dass wir die Ladder-Operatoren bereits definiert haben a und a und haben einen "Grundzustand" definiert | 0 (wir haben immer noch nicht bewiesen, dass es der Grundzustand ist) so dass a | 0 = 0 . Wir gehen auch davon aus, dass wir bereits wissen, dass der Hamiltonoperator des harmonischen Oszillators in der Form geschrieben werden kann

H = ω ( a a + 1 2 ) .

(Beachten Sie, dass der Grundzustand | 0 ist trivialerweise ein Eigenzustand mit E 0 = ω / 2 .) Abschließend nehme ich an, dass wir bereits die Kommutierungsrelationen der Leiteroperatoren gezeigt haben. Nämlich,

[ a , a ] = 1.

Damit haben wir genug für einen Beweis.

Wir können einen Zustand definieren | n (vergessen wir vorerst die Normalisierung) as

| n = ( a ) n | 0 ,

wo n ist eine nichtnegative ganze Zahl. Der Staat | n ist ein Eigenzustand des Hamiltonoperators mit Energie E n = ω ( n + 1 / 2 ) . Wir möchten zeigen, dass das Set { | n } n Z + sind alle möglichen normierbaren Eigenzustände des Hamiltonoperators.

Erinnern Sie sich an die Positionsdarstellung, wenn wir ein Potenzial haben v ( x ) , dann können wir keinen normierbaren Eigenzustand haben | ψ dessen Energie erfüllt E ψ Mindest v ( x ) . Das heißt, wir können keine Energie haben, die kleiner ist als die minimale potentielle Energie des Systems (dh die kinetische Energie muss positiv sein).

Nun schließen wir mit einem Widerspruchsbeweis ab. Betrachten Sie einen Eigenzustand | ψ dessen Energie gegeben ist durch E ψ = ω ( n + 1 / 2 + ϵ ) , mit ϵ ( 0 , 1 ) . Ein solcher Zustand würde im Wesentlichen jeden der "anderen" Zustände beschreiben H zulassen könnte. Betrachten Sie nun den Staat | ψ ( 1 ) = a | ψ . Mit der Kommutatoralgebra ist das nicht schwer zu zeigen | ψ ( 1 ) hat Energie

E ψ ( 1 ) = ω ( ( n 1 ) + 1 2 + ϵ ) .

Nun können wir einen Zustand einführen und definieren | ψ ( m ) ( a m ) | ψ . Offensichtlich wird seine Energie durch gegeben

E ψ ( m ) = ω ( ( n m ) + 1 2 + ϵ ) .

Sofern dieser Prozess also nicht irgendwann beendet wird (d. h. a | ψ ( m ) = 0 für einige m ) können wir eine beliebig niedrige Energie erreichen. Allerdings könnte dieser Prozess nie enden, da der Grundzustand | 0 ist eindeutig (er ist in Bezug auf einen Positionsoperator und einen einzelnen Ableitungsoperator definiert, also a | 0 = 0 definiert einfach eine Differentialgleichung erster Ordnung im Ortsraum) und hat Energie ω / 2 , dies kann für niemanden erreicht werden ϵ im angegebenen Bereich. Also kein solcher Zustand | ψ kann auftreten. Ebenso können wir keinen Zustand mit Energie haben E ψ ( 0 , ω / 2 ) nach der gleichen Logik.

Damit haben wir (sehr rigoros) gezeigt, dass die einzig normierbare von H sind diejenigen mit Energie ω ( n + 1 / 2 ) , die ausschließlich durch die Wirkung von Leiteroperatoren auf den Grundzustand erzeugt werden.

Ich hoffe, das hat geholfen!

(TL;DR – Wenn ein anderer Zustand existieren würde, hätte er eine Energie, die nicht der Form entspricht, die von Leiteroperatoren gegeben wird. Allerdings wirkt er viele Male auf diesen Zustand ein a eine beliebig niedrige Energie erzeugen würde und somit ein solcher Zustand nicht existieren könnte.)

Ich mag diesen Beweis, da er intuitiv sinnvoll ist und die Frage beantwortet. Ist dieser Beweis jedoch nicht technisch unnötig, da wir wissen, dass das Spektrum von H besteht aus ω ( n + 1 / 2 ) für n>=0, und zeigen Sie, wie wir jeden Eigenzustand erreichen, indem wir den Erzeugungsoperator verwenden, der auf den Grundzustand einwirkt? Ich möchte die Frage oder Antwort nicht schmälern, ich denke, sie sind beide sehr gut.
Es geht darum zu beweisen, dass die Energien, die Sie gegeben haben, die einzigen sind, die für dieses Problem möglich sind.
@BobKnighton (1/3) Ich denke, dass einer der Schwachpunkte in diesem Beweis darin besteht, dass Sie hier keine allgemeine Eindeutigkeit des Grundzustands bewiesen haben, mit Ausnahme (erneut) des traditionellen Leiteroperators ( a ) und seine Befugnisse ( a m ), deren Eindeutigkeit zunächst fraglich ist. Ja, a | 0 >= 0 definiert eine Differentialgleichung 1. Ordnung, jedoch für Operatoren a . Ein weiterer Punkt ist, dass selbst nach dem Beweis der Eindeutigkeit des Grundzustands für solche Operatoren (nicht schwierig, siehe Beweis hier: physical.stackexchange.com/questions/380655/… ) …
@BobKnighton (2/3) … es gibt eine zweite inhärente Annahme in Ihrem Beweis, dass die a Operatoren sind diejenigen, die immer die kleinsten Schritte erzeugen würden ( ω ) und dienen somit als Vergleichsmittel. Machen Sie daher Schritte nach oben/unten mit der gleichen Auflösung, die durch definiert ist a ist verpflichtet, alle anderen Ebenen abzulehnen. Aber wenn eine feinere Auflösung erzeugt werden könnte durch …
@BobKnighton (3/3) … ein weiterer Leiteroperator, die traditionellen Operatoren a wäre damit nicht kompatibel. (Beachten Sie, dass anstatt den Punkt für additiv zu beweisen ϵ , man muss es für multiplikative Faktoren wie beweisen ( 1 ± ϵ ) ω stattdessen, um die Operatoren anzusprechen). Was großartig zu sehen wäre, ist ein rigoroser Beweis dafür, dass kein anderer Operator einen kleineren Schritt machen könnte. Dies wurde kürzlich in diesem Beitrag gefragt, und es wäre schön, wenn Sie Ihre Erkenntnisse hinzufügen könnten: physical.stackexchange.com/q/380655

Um dies zu beweisen, müssen Sie den folgenden Satz kennen: Es gibt keine degenerierten gebundenen Zustände in 1D . Der Beweis ist im Link skizziert. Beim Oszillatorpotential ist jeder Eigenzustand ein gebundener Zustand, es gibt also keine Entartung im Spektrum.

Sobald wir das wissen, ist der Rest des Beweises ziemlich einfach. Im Folgenden ignoriere ich Normalisierung und set ω = 1 um die Notation zu erleichtern. Wir haben einen eindeutigen Grundzustand, gegeben durch | 0 , was befriedigt a | 0 = 0 , und H | 0 = 1 2 | 0 . Auf diesem Grundzustand definieren wir einen Turm von Zuständen | n = ( a ) n | 0 , die befriedigen H | n = ( n + 1 2 ) | n . Ich behaupte, dass dies alle Eigenzustände sind. Wir werden es durch Widerspruch beweisen.

Angenommen, es gibt einen anderen Eigenzustand | ϕ das gehört nicht dazu | n . Dann H | ϕ = E ϕ | ϕ . Unter Verwendung der Kommutierungsrelation von a und H , das kannst du zeigen a k | ϕ ist auch ein Eigenzustand des Hamiltonoperators mit Eigenwert E ϕ k . Da wir wissen, dass die Eigenwerte nicht negativ sein können, bedeutet das für einige k dieser Prozess muss beendet werden. Mit anderen Worten, für einige k gilt a k | ϕ 0 aber a k + 1 | ϕ = 0 . Aber wenn das stimmt, a k | ϕ hat zwangsläufig Energie 1 2 , seit H a k | ϕ = ( a a + 1 2 ) a k | ϕ = 1 2 | ϕ . Aber wir wussten auch, dass dieser Zustand Energie hatte E ϕ k . Daher, E ϕ k = 1 2 , oder E ϕ = k + 1 2 .

Aber das ist ein Widerspruch! Das bedeutet, dass | ϕ und | k die gleiche Energie haben. Da es also in 1D keine Entartung gibt, müssen sie den gleichen Zustand haben. Dies widerspricht unserer Annahme, dass | ϕ war nicht in unserer ursprünglichen Liste der Staaten.


EDIT: Als Antwort auf Ihre Verwirrung in den Kommentaren würde vielleicht ein konkretes Beispiel helfen. Nehmen wir an, es gäbe einen Staat | 1 2 . Dann a | 1 2 wäre ein Staat | 1 2 , und a 2 | 1 2 wäre ein Staat | 3 2 , usw. Dies ist nicht erlaubt, weil wir wissen, dass unser Hamiltonoperator nur nicht-negative Eigenvektoren hat, aber z H | 3 2 = ( 1 ) | 3 2 .

Die einzige Möglichkeit, keine willkürlich negativen Eigenvektoren zu erhalten, besteht darin, irgendwann a k | ϕ kein gültiger Zustand Ihres Systems mehr ist. Was ist ein ungültiger Zustand des Systems? Der Nullvektor ist der einzige Vektor im Hilbert-Raum, der kein erlaubter Zustand Ihres Systems ist. Irgendwann muss man also haben a k + 1 | ϕ = 0 . Aber, wie ich oben argumentiert habe, haben a k + 1 | ϕ = 0 impliziert, dass a k | ϕ hat zwangsläufig Energie 1 2 . Das bedeutet, dass | ϕ Energie hatte k + 1 2 , also ist | k . Das bedeutet es auch a k | ϕ ist eigentlich nur | 0 , seit | 0 ist der einzigartige Vektor mit Energie 1 2

Nirgendwo haben wir davon ausgegangen | ϕ war kein Staat wie | 1.41 . Aber wir haben bewiesen , dass wir, wenn es einen solchen Zustand gäbe, Leiteroperatoren verwenden könnten, um eine ganze Reihe von Zuständen mit beliebig negativen Energien zu erzeugen. Da wir wissen, dass unser Hamiltonoperator keine negativen Energiezustände hat, impliziert dies, dass ein Eigenzustand wie | 1.41 kann nicht existieren.

Wenn Sie schreiben: „Mit anderen Worten, für einige k, a k | ϕ 0 aber a k + 1 | ϕ = 0 .' , davon gehst du aus a k + 1 | ϕ ist der Grundzustand. Es kann nicht so sein. Ich meine, diese Annahme ist nicht gerechtfertigt. Wenn Sie diesen Zustand als Grundzustand betrachten, wird das Problem selbst gelöst. Das mag sein a k + 1 | ϕ ist ein Zustand, der sozusagen zwischen dem Grundzustand und dem ersten angeregten Zustand liegt | 1 2 .
Das ist der Kern meiner Frage. Ich möchte wissen, ob es Energieniveaus oder solche Energiezustände gibt | π , | 1.57 , | 2.31 zwischen Zuständen wie | 1 , | 2 , | 3 usw. Und der Name dieser Zustände ist nicht nur für irgendeine komische Nomenklatur gedacht, sondern ich möchte mit dieser Benennung Zustände bezeichnen, die tatsächlich dazwischen liegen | n wobei n eine ganze Zahl ist. Oder besser gesagt, die Energiezustände, die mit dem Ladder-Operator nicht gefunden werden können.
@SchrodingersCat Davon gehe ich überhaupt nicht aus! a k + 1 | ϕ ist kein Grundzustand. Es ist überhaupt kein Staat. es ist 0 . Nicht | 0 , aber in Wirklichkeit 0 , wie im Nullvektor.
Das einzige, was ich annehme (und das kann man auch beweisen), ist, dass es keine Eigenzustände mit negativer Energie gibt. Das bedeutet, dass nicht jeder a k | ϕ kann ein Eigenzustand sein, da für hinreichend groß k , E ϕ k wird negativ sein. Das heißt irgendwann a k + 1 | ϕ muss für einige identisch Null sein k , da es sonst ein Eigenzustand wäre.
Denk daran, dass 0 ist kein gültiger Zustand für ein Quantensystem. Es ist nur der Nullvektor in unserem Hilbert-Raum.
@SchrodingersCat Ich habe meine Antwort bearbeitet, lass es mich wissen, wenn es immer noch Verwirrung gibt. Das hat mich auch gestört, als ich zum ersten Mal von der SHO erfahren habe, daher würde ich gerne näher darauf eingehen.
@JahanClaes (1/2) Ja, aber ich denke, die wichtigste tiefere Frage betrifft die Einzigartigkeit dieser Leiteroperatoren im Allgemeinen. Ich denke, Sie machen in Ihrem Beweis eine inhärente Annahme, dass die traditionellen Operatoren a sind diejenigen mit der kleinstmöglichen Schrittweite ( ω oder 1 oben) und daher messen Sie alles an ihnen als Haupt-"Einheiten" für andere Betreiber; daher Ihre Beschränkung auf Operatoren des Formulars a k mit k Z (z. B. deshalb deine H a k | ψ > gab ( 1 / 2 ) | ψ > in erster linie, da arbeitest du noch ab a als Baustein). Aber…
@JahanClaes (2/2)… was wäre, wenn andere Operatoren (möglicherweise entsprechend gebrochenen Potenzen von a , oder sonst) wurden verwendet und ergaben Kommutatorwerte <1? Sie zitieren auch die Nichtentartung, aber auch hier ist die Entartung nur für einen bestimmten Operator von Bedeutung, aber was ist, wenn es um andere mögliche Operatoren geht, die eine andere Leiter erzeugen? Eine ähnliche Frage wurde kürzlich hier gestellt, es wäre schön, wenn Sie Ihre Erkenntnisse dazu hinzufügen könnten: physical.stackexchange.com/questions/380655/…
  1. Hier nehmen wir an, dass der harmonische Quantenoszillator in einer algebraischen (im Gegensatz zu einer geometrischen) Einstellung gegeben ist. Sag, dass wir nur das wissen

    (1) H ^ ω   :=   N ^ + v 1 , v R ,
    (2) N ^   :=   a ^ a ^ ,
    (3) [ a ^ , a ^ ]   =   1 , [ 1 , ]   =   0.
    Auch nehmen wir an, dass die physikalischen Zustände in einem inneren Produktraum leben ( v , , ) .

  2. In meiner Phys.SE-Antwort hier wurde mit diesen Annahmen dann gezeigt, dass das Punktspektrum der Zahlenoperator ist N ^ ist genau alle nicht negativen ganzen Zahlen

    (4) S p e c p ( N ^ )   =   N 0 .

  3. Insbesondere ist es möglich, alle (möglicherweise entarteten) Energieniveaus zu erreichen, indem man mit dem Erzeugungsoperator auf einen Vakuumzustand einwirkt a ^ .

  4. Bei der algebraischen (im Gegensatz zu einer geometrischen) Einstellung gibt es jedoch eine Einschränkung: Der Vakuumzustand muss nicht eindeutig sein!

  5. Beispiel:

    (5) v   =   F EIN F B
    könnte eine direkte Summe zweier Fock-Räume mit Vakuumzuständen sein | 0 EIN und | 0 B , beziehungsweise. In diesem System ist ein allgemeiner Vakuumzustand eine Linearkombination von | 0 EIN und | 0 B . Beachten Sie, dass es dann nicht möglich wäre, einen bestimmten festen Vakuumzustand in einen beliebigen Energiezustand umzuwandeln, indem Sie nur mit dem Erzeugungsoperator agieren a ^ . Wir brauchen auch die anderen Vakuumzustände.

Nehmen wir das an | m ist ein Eigenzustand von N = a a , dh N | m = m | m , und das m N .

Dann seit

a | m = c | m 1

mit c C , für einige k N wir werden haben

a k | m = c ' | m k

mit

m k < 0

Wir werden jetzt beweisen, dass dies ein Absurdum ist . In der Tat, wenn ein Eigenwert gegeben ist n von N , wir haben

n = n | N | n = n | a a | n = a | n 2 0

Wir schließen dann diese Hypothese ab m N muss falsch sein, und das m muss eine nicht negative Ganzzahl sein.